Weakening Argument

This topic has expert replies
Master | Next Rank: 500 Posts
Posts: 199
Joined: Mon Apr 06, 2009 4:15 am
Location: India
Thanked: 13 times

Weakening Argument

by gauravgundal » Thu Jul 07, 2011 4:40 am
Source Manhattan Foundation for GMAT Verbal

Ostrich meat,which is low in fat and cholesterol,is becoming popular in the united states among dieters and those concerned with health. However,ostriches are an endangered species. If the popularity of ostrich meat continues to grow at the present rate ,ostriches will inevitably become extinct.
Which of the following ,if true most seriously weakens the argument above?
A. Ostrich meat is not any healthier than lean beef,which is available everywhere and less expensive.
B. Ostrich meat is considered by many to be a gourmet food,suitable only for special occasions.
C. Concerns about avian flu,although spurious,have caused many consumers to avoid ostrich meat.
D. An increased demand for ostrich meat will lead to increased ostrich farming ,thus increasing the number of ostriches.
E. Ostriches are killed more frequently for their feathers for the making of leather than they are for meat.

[spoiler]OA: D[/spoiler]
I do agree with answer choice D,but initially I end up selecting answer choice E.

Reason for selecting answer choice E.
Assumption of the argument is if the popularity of ostrich meat increases then rate of killing ostrich will be high.

Since answer choice E weakens he assumption,stating that the reason for killing the ostrich is not absolutely meat.

After writing this I do feel that answer choice E still supports the conclusion that ostrich will become extinct.

Am I wrong in my methodology of weakening the argument ,that is ,I am weakening the assumption of the argument ,which in turn, should weaken the argument.

User avatar
Legendary Member
Posts: 1325
Joined: Sun Nov 01, 2009 6:24 am
Thanked: 105 times
Followed by:14 members

by vikram4689 » Thu Jul 07, 2011 5:11 am
After writing this I do feel that answer choice E still supports the conclusion that ostrich will become extinct.
you have to weaken the conclusion of the argument. And E does not weaken the argument, weakening means proving that even after killing increase,ostrich would not become extinct
Premise: If you like my post
Conclusion : Press the Thanks Button ;)

User avatar
GMAT Instructor
Posts: 15539
Joined: Tue May 25, 2010 12:04 pm
Location: New York, NY
Thanked: 13060 times
Followed by:1906 members
GMAT Score:790

by GMATGuruNY » Thu Jul 07, 2011 6:47 am
-
gauravgundal wrote:Source Manhattan Foundation for GMAT Verbal

Ostrich meat,which is low in fat and cholesterol,is becoming popular in the united states among dieters and those concerned with health. However,ostriches are an endangered species. If the popularity of ostrich meat continues to grow at the present rate ,ostriches will inevitably become extinct.
Which of the following ,if true most seriously weakens the argument above?
A. Ostrich meat is not any healthier than lean beef,which is available everywhere and less expensive.
B. Ostrich meat is considered by many to be a gourmet food,suitable only for special occasions.
C. Concerns about avian flu,although spurious,have caused many consumers to avoid ostrich meat.
D. An increased demand for ostrich meat will lead to increased ostrich farming ,thus increasing the number of ostriches.
E. Ostriches are killed more frequently for their feathers for the making of leather than they are for meat.

[spoiler]OA: D[/spoiler]
I do agree with answer choice D,but initially I end up selecting answer choice E.

Reason for selecting answer choice E.
Assumption of the argument is if the popularity of ostrich meat increases then rate of killing ostrich will be high.

Since answer choice E weakens he assumption,stating that the reason for killing the ostrich is not absolutely meat.

After writing this I do feel that answer choice E still supports the conclusion that ostrich will become extinct.

Am I wrong in my methodology of weakening the argument ,that is ,I am weakening the assumption of the argument ,which in turn, should weaken the argument.
I received a PM asking me to comment.

You are misidentifying the assumption.
This argument exhibits a language shift.
The premise is that ostriches are becoming increasingly popular.
The conclusion is that ostriches will become extinct.
The assumption is that these two ideas are connected: that popularity leads to extinction.

Answer choice E discusses only the premise: WHY the ostriches are popular. It does not attack the assumption that popularity is linked to extinction. To weaken the conclusion, the correct answer must break this link: it must show that, even if ostriches become more popular, they will NOT become extinct.

Answer choice D does just what we need: An increased demand for ostrich meat will lead to increased ostrich farming, thus increasing the number of ostriches. An increase in the number of ostriches means that, even if ostriches become more popular, they will NOT become extinct.
Private tutor exclusively for the GMAT and GRE, with over 20 years of experience.
Followed here and elsewhere by over 1900 test-takers.
I have worked with students based in the US, Australia, Taiwan, China, Tajikistan, Kuwait, Saudi Arabia -- a long list of countries.
My students have been admitted to HBS, CBS, Tuck, Yale, Stern, Fuqua -- a long list of top programs.

As a tutor, I don't simply teach you how I would approach problems.
I unlock the best way for YOU to solve problems.

For more information, please email me (Mitch Hunt) at [email protected].
Student Review #1
Student Review #2
Student Review #3

Master | Next Rank: 500 Posts
Posts: 138
Joined: Mon Mar 14, 2011 8:24 pm
Thanked: 1 times

by rishijhawar » Mon Jul 11, 2011 5:56 am
+1 for D.
I think one of the most efficient/straightforward ways to answer a weaken question is to ATTACK the conclusion HARD(I strongly believe most of us do that).
Here the conclusion is Ostrich will become extinct. So we need to find an choice which will say that Ostrich will not become extinct despite its popularity. Only D helps to zero-in this.
Hence correct answer is D.

Legendary Member
Posts: 1404
Joined: Tue May 20, 2008 6:55 pm
Thanked: 18 times
Followed by:2 members

by tanviet » Tue Jul 12, 2011 5:39 am
The argument is simple. The hard thing is C choice which is a trap. This trap affects conclusion outside context of evidence.

There are a lot of this traps on GMATPREP tests.